Vous êtes sur la page 1sur 42

Version 022 Test 4 florin (57850)

This print-out should have 20 questions.


Multiple-choice questions may continue on
the next column or page find all choices
before answering.
001 10.0 points
A mechanical lift is used to raise a heavy load.
The lift consists of a horizontal platform of
length L drawn by vertical cables on either
end of the platform. The load is placed on
the platform and drawn slowly upward at a
constant velocity so that acceleration is not
important. The platform has a mass m. The
load has mass 4m and is placed a distance L/4
from the right end of the lift. The acceleration
due to gravity is g. What is the tension in the
right vertical cable?
1. 1/3 mg
2. 7/2 mg correct
3. 5/4 mg
4. mg

the Mississippi River and deposited in the


Gulf of Mexico each year. Thus, there is a
net movement of mass southward towards the
equator.
What effect does this tend to have on the
length of a day?
1. Impossible to determine
2. shorten the day
3. lengthen the day correct
4. no change
Explanation:
Soil washed down the river is deposited at
a greater distance from the Earths rotational
axis. Just as a man on a turntable slows
down when one of the masses is extended, the
Earth slows down in its rotational motion,
extending the length of the day. The amount
of slowing, of course, is exceedingly small.
(Interestingly, the construction of many dams
in the Northern hemisphere has the opposite
effect; shortening our days!)

5. 2/3 mg
003 10.0 points
A length of light nylon cord is wound around
6. 3/2 mg
a uniform cylinder of radius 0.735 m and mass
1.36 kg. The cylinder is mounted on a fric7. 7/4 mg
tionless axle and is initially at rest. The cord
is pulled from the cylinder with a constant
8. 3 mg
force of magnitude 5.43 N.
How fast will the spool be rotating after the
9. 4/5 mg
string has been pulled for t = 3.63 s?
The moment of inertia of the cylinder is
10. 2/7 mg
1
I = m r2.
Explanation:
2
The net torque about any axis must be zero.
1. 145.904
Since we want the tension in the right cable,
2. 51.2494
compute the net torque about the left end:
3. 31.6695
4. 189.224
X
5. 113.85
3
1
L = L(4 mg) L(mg)+L(TR )+0(TL ) = 0. 6. 44.2199
4
2
7. 24.9011
Therefore TR = 7/2 mg.
8. 11.3012
9. 39.4376
002 10.0 points
10. 26.338
A sizable quantity of soil is washed down

Version 022 Test 4 florin (57850)


Correct answer: 39.4376 rad/s.

What is the new speed of the puck?

Explanation:
Let :

1. v2 = 2 v1 correct
F = 5.43 N ,
m = 1.36 kg ,
r = 0.735 m ,
t = 3.63 s .

2. v2 = 0
and

The angular momentum after 3.63 s is


L = t
= F r t
= (5.43 N)(0.735 m)(3.63 s)
= 14.4875 kg m2 /s .
The moment of inertia of the cylinder is


1
2
mr
=
2
1
= (1.36 kg) (0.735 m)2
2
= 0.367353 kg m2 .
Therefore, the rotational velocity will be
= L/I
= (14.4875 kg m2 /s)/(0.367353 kg m2 )
= 39.4376 rad/s .
004 10.0 points
A small puck moves in a circle on a frictionless
airtable. The circular motion is enforced by
string tied to the puck and going through a
tiny hole in the middle of the table. Initially,
the puck moves in a circle of radius R1 at
speed v1 . But later the string is pulled down
through the hole forcing the puck to move in
1
a smaller circle of radius R2 = R1 .
2

1
3. v2 = v1
2
4. v2 = v1
1
v1
2
1
6. v2 = v1
2 2

7. v2 = 2 2 v1

5. v2 =

8. v2 =

1
v1
4

9. v2 = 4 v1

10. v2 = 2 v1
Explanation:
Let the hole in the airtable be the origin
of our coordinate system. Because the hole
is tiny, the string always pulls the puck in
the radial direction. Consequently, the string
~ has zero torque (about the
tension force T
origin). The other two forces on the puck
~ and the normal force N
~ of the
the weight W
table cancel each other and each others
torques. Altogether, we have zero net torque,
so the angular momentum of the puck must
be conserved:
~ =R
~ m~v = const.
L
When the puck moves in a circle, the direction
of the angular momentum is vertically up, and
its magnitude is L = m v R . This is true both
before and after the string being pulled down,
so

R
v

L = m v1 R1 = m v2 R2
R1
v2 =
v1 = 2 v1 .
R2
005

10.0 points

Version 022 Test 4 florin (57850)


A rod has a pivot at one end and is free to
rotate without friction at the other end, as
~ is applied to the free end
shown. A force F
at an angle to the rod creating a torque ~
about the pivot.
L

Which tires have the greater rotational


speed?
1. It depends on which tires have more
mass.
2. It depends on the angular acceleration of
the wheels.

3. Beths tires correct

4. Harrys tires
If instead the same force is applied perpendicular to the rod, at what distance d from the
pivot should it be applied in order to produce
the same net torque ~ about the pivot?

5. It depends on which tires have more angular momentum.


6. The rotational speeds are the same.

1. d = L/ tan

2. d = 5 L

7. It depends on the which tires have a larger


moment of inertia.

3. d = L cos

8. It depends on the center of mass speed.

4. d = L/ sin

Explanation:
v = r . Tires with a smaller radius needs
a larger rotational speed to obtain the same
linear speed.

5. d = L sin correct
6. d = L/ cos

007 10.0 points


A uniform meter-stick with length L pivots
at point O. The meter stick can rotate freely
about O and is released from the horizontal
position at t = 0.

7. d = L
8. d = L tan

9. d = 2 L

10. d = L/2
Explanation:
The force generates a torque of
= F L sin ,
so the distance is L sin .
006 10.0 points
Harry and Beth cycle at the same speed, i.e.
their centers of mass move with the same
velocity. The bike tires all rotate without
slipping, but the tires on Harrys bike have a
larger radius than those on Beths bike.

Determine the angular acceleration of the


meter stick at the moment it is released.
5g
6L
3g
2.
4L
5g
3.
4L
1.

Version 022 Test 4 florin (57850)


g
2L
g
5.
4L
g
6.
3L
7g
7.
4L
g
8.
L
3g
9.
correct
2L
2g
10.
L
Explanation:
The equation of motion = I gives:
mgL
1
= I = mL2 ,
2
3
So
3g
=
.
2L
4.

008 10.0 points


A flywheel with a very low friction bearing
takes 1.7 h to stop after the motor power
is turned off. The flywheel was originally
rotating at = 3.14159 rad/s.
Assuming the angular deceleration is constant, how many revolutions does the flywheel
make before it stops? Remember that the
number of revolutions is given by the /2.
1. 2310.0
2. 2484.0
3. 2250.0
4. 1617.0
5. 2205.0
6. 2829.0
7. 3192.0
8. 3960.0
9. 1530.0
10. 2760.0
Correct answer: 1530 rev.
Explanation:
Let : t = 1.7 h .
The angular acceleration of the flywheel is
0
=
,
t

so the angle through which it rotates before


stopping is given by



1
1 0
2
= 0 t + t = 0 t +
t2
2
2
t
1
= 0 t
2
3600 s 1 rev
1
= (3.14159 rad/s) (1.7 h)
2
1h
2
= 1530 rev .

009

10.0 points

Consider an isolated system with 100


quanta of energy distributed between two
blocks in contact. Block 1 has 300 quantum
mechanical harmonic oscillators and Block 2
has 200 oscillators. The figure shows plots of
entropy of block 1 S1 and entropy of block 2
S2 versus quanta q. Note that the entropy of
block 2 is plotted with its x-axis reversed so
that m2 has a positive upward slope in the
figure.
Initially, there are 90 quanta in block one
and 10 quanta in block 2, but the system is
not in equilibrium. At equilibrium, there are
60 quanta in block 1 and 40 quanta in block
2. The initial slopes of the entropy curves
are indicated in the figure as m1 and m2 ,
respectively. After the system reaches equilibrium, what will be the relationship between
the slopes of the two curves.

Version 022 Test 4 florin (57850)


1. more information is needed
2. m1 =

1
m2
2

3. m1 = m2 = 0
4. m1 = m2 =
5. m1 = m2 correct
6. m1 < m2

5



20!
9. S = kB ln
7! 5!


21!
10. S = kB ln
7! 5!
Explanation:
Entropy is defined as S = kB ln where
is the number of possible microstates, which
is the number of ways to arrange q quanta in
n harmonic oscillators, i.e.

7. m1 > m2
=

8. the slopes do not change


Explanation:
The slope of the entropy curve gives the
temperature of each block with a given quanta
of energy in that block. When equilibrium is
reached, the temperatures will be the same
so the slopes should be the same. However,
the axis for block 2 is reversed so the correct
answer is m1 = m2 .
010 10.0 points
Using the Einstein model of a solid, what
is the change in entropy when adding two
quanta of energy to a system of 5 atoms that
already has 5 quanta of energy stored in it?


21
1. S = kB ln
75


16!
2. S = kB ln
7! 7
 
10
3. S = kB ln
7
 
17
4. S = kB ln
6
5. S = kB ln (10) correct
 
10!
6. S = kB ln
7!


21!
7. S = kB ln
14! 7!


20!
8. S = kB ln
14! 5!

(n 1 + q)!
(n 1)! q!

Here, n 1 = 3 5 1 = 14 and initially


qi = 5, but in the final state qf = 7. Thus,
the change in entropy is

S = Sf Si




(14 + 7)!
(14 + 5)!
= kb ln
kb ln
14! 7!
14! 5!


= kb ln 

(14+7)!
14!7!

(14+5)!
14!5!


21 20
= kb ln
76


21 10
= kb ln
73
= kb ln (10)


011 10.0 points


Two masses of 22 kg and 11 kg are suspended
by a pulley that has a radius of 15 cm and
a mass of 6 kg. The cord has a negligible
mass and causes the pulley to rotate without
slipping. The pulley rotates without friction
1
its moment of inertia is given by I = M R2.
2

Version 022 Test 4 florin (57850)

m2
m2 (+a) = T2 m2 g
T2 = m2 g + m2 a

15 cm

6 kg

So far we have two equations and three unknown: T1 , T2 , and a. However, the Angular
Momentum Principle gives us another equation

I = net
= T1 R T2 R

22 kg

11 kg
Determine the angular acceleration of the
pulley after the masses are released and before
they fall off of the pulley.
The acceleration of gravity is 9.8 m/s2 .
1. 42.2323
2. 36.3367
3. 29.4
4. 27.2222
5. 23.038
6. 14.2029
7. 62.5324
8. 38.8632
9. 19.963
10. 21.9103
Correct answer: 19.963 rad/s2 .
Explanation:
Let : M = 6 kg ,
R = 15 cm ,
m1 = 22 kg , and
m2 = 11 kg
The tension in the cord attached to the
first mass T1 is different than the tension in
the cord attached to the second mass T2 . We
must use both the Momentum Principle and
the Angular Momentum Principle to solve the
problem.
The Momentum Principle for m1 gives:
m1 (a) = T1 m1 g
T1 = m1 g m1 a
where we have taken care to make the acceleration negative since it will be positive for

where again we have taken care to get the


signs of alpha and the torque correct. We can
now plug in the expressions for tension while
making the substitution that a = R since
there the cord does not slip.
I = (m1 g m1 R)R (m2 g + m2 R)R
= +(m1 m2 )gR (m1 R2 + m2 R2 )
= [(m1 m2 )gR]/(I + m1 R2 + m2 R2 )
= 19.963 rad/s2 .
012 (part 1 of 2) 10.0 points
A mass m is located at a distance R from
the center of a planet of mass M . The initial
speed of m is v0 and its velocity vector makes
an angle with respect to the line joining the
mass and the center of the planet. You can
assume that the planet remains stationary in
course of the motion of the projectile.
Find the magnitude of the angular momentum of the mass m relative to the center of
the plant.
1. M v0 R cos
2. mv0 R sin + M v0 R sin
3. mv0 R sin M v0 R sin
4. mv0 R cos M v0 R cos
5. 0
6. M v0 R sin

Version 022 Test 4 florin (57850)

7. mv0 R cos

1. 10

8. mv0 R sin correct

2. cannot be determined

Explanation:
The magnitude of the angular momentum
vector,
~ m | = |~r ~p| = mv0 R sin .
|L
~r is the position of the mass m w.r.t. center
of the planet and ~p is the momentum vector
of the mass.

3. 15
4. 1 1015
5. 225
6. 150 correct
7. 25

013 (part 2 of 2) 10.0 points


What is the magnitude of the net torque on
the mass m about the center of the planet?
1. Need more information
2. GmM cos /R

Explanation:
The total number of microstates or number of ways of arranging energy in the system
is the product of the number of ways of arranging the energy in respective objects, i.e.
total = 1 2
015

3. 0 correct

A force

4. GmM cos /R2


5. GmM sin /R
6. GmM sin /R2
7. GmM/R
Explanation:
The net torque on the mass m due to the
gravitational attraction of the planet,
~ grav .
~ = ~r F
~ grav are anti-parallel at all times,
Since ~r and F
~ = 0 always.
014 10.0 points
Two objects share a total energy E = E1 +E2 .
There are 10 ways to arrange an amount of
energy E1 in the first object and 15 ways to
arrange an amount of energy E2 in the second
object. How many different ways are there to
arrange the total energy E = E1 + E2 so that
there is E1 in the first object and E2 in the
other?

10.0 points

~ = (3 + 6) N
F

acts at a distance from the point of rotation


given by
~r = (8 + 4) m,
where and are unit vectors pointing in the
+x and +y directions respectively. What is
the torque about the point of rotation created
by this force? (Note that k is a unit vector
pointing in the +z direction.)
1. (50 + 60) N m
N m correct
2. (60k)
Nm
3. (14 + 50k)
4. (60) N m
5. (50 36) N m
Nm
6. (60 + 50k)
Nm
7. (60k)
8. (36) N m

Version 022 Test 4 florin (57850)


Nm
9. (36k)
Nm
10. (14 50k)

Explanation:
~ . Because both
Torque is defined as ~r F
vectors are in the xy plane, their cross product
must be in the z direction. So we only need
to look at the z-component of the cross product. The z component of the cross product
will be
rx Fy ry Fx = 60 N m.
Nm .
So the resultant vector is (60k)
016 10.0 points
A ball of putty with mass m falls vertically
onto the outer rim of a turntable of radius
R and moment of inertia I0 that is rotating
freely with angular speed i about its vertical
fixed symmetry axis, i.e. the turntable is
horizontal.
What is the post-collision angular speed of
the turntable plus putty?
1. f =
2. f =
3. f =
4. f =
5. f =
6. f =
7. f =
8. f =

i
m R4
2+
I0
i
m R3
3m +
I0
i
mR
1+
I0
i
m R2
2+
I0
i
m R3
2m +
I0
i
mR
2+
I0
i
m R3
1+
I0
i
m R3
m+
I0

i
m R4
1+
I0
i
10. f =
correct
m R2
1+
I0
Explanation:
The final rotational inertia of the turntableplus-putty is
9. f =

If = I0 + Iblob = I0 + m R2 .
Since there is no external torque on the system
of the putty plus the turntable, we know Lf =
L i = I 0 i .
I f f = I 0 0
I 0 i
f =
If
I 0 i
f =
I0 + m R 2
i
=
.
m R2
1+
I0
017 10.0 points
In the figure, two objects of the same mass
m = 2.1 kg are connected by a massless rod
of length d = 1.25 m. At a particular instant
they have velocity magnitudes that are v1 =
32 m/s and v2 = 79 m/s, respectively. The
system is moving in outer space far from any
other objects. The x direction is to the right,
y is up, and z is out of the page toward you.
m
b

v1

b
v2
m
What is the magnitude of the rotational
angular momentum Lrot of the system?
1. 42.0

Version 022 Test 4 florin (57850)


2. 61.6875
3. 22.3125
4. 107.625
5. 70.875
6. 87.9375
7. 86.625
8. 93.1875
9. 63.0
10. 59.0625
Correct answer: 61.6875 kgm2 /s.
Explanation:
The rotational angular momentum is the
angular momentum of the system about its
center of mass. As the objects are of equal
mass, the center of mass of the system is
halfway between them. Summing the angular
momentum of each mass,

rotational inertia about an axis through the


1
center of mass is ICM =
M 2 .
12
2
1. L = M 2 .
3
5
2. L = M 2 .
4
5
3. L =
M 2 .
12
11
M 2 .
4. L =
3
13
5. L =
M 2 .
12
9
M 2 .
6. L =
12
7. L = M 2 .
11
M 2 .
12
1
9. L = M 2 .
3
17
10. L =
M 2 . correct
12
8. L =

~ rot = ~r1,CM ~p1 + ~r2,CM ~p2


L
d
= h0, , 0i hm v1 , 0, 0i
2
d
+ h0, , 0i hm v2 , 0, 0i
2
Explanation:
the parallel axis theorem,
= h0, 0, (0.625 m)[(2.1 kg)(79 m/s) (2.1 kg)(32Using
m/s)]i
= h0, 0, 61.6875 kg m2 /si .

The magnitude is thus 61.6875 kg m2 /s .


018 10.0 points
Two rigid rods of length and mass M are
rigidly attached as shown

pivot
What is the magnitude of the angular momentum L of this system when it is rotating at
an angular velocity about an axis through
the end of one rod, as indicated in the sketch?
The rotational inertia of a rod about an axis
1
through one end is I =
M 2 , while the
3

1
1
M 2 +
M 2 + M 2
3
12


4 + 1 + 12
17
=
M 2 =
M 2 .
12
12

I=

Therefore, the angular momentum is


L = I =

17
M 2
12

019 10.0 points


Consider a uniform ladder leaning against
a smooth wall and resting on a smooth floor
at point P . There is a rope stretched horizontally, with one end tied to the bottom of the
ladder essentially at P and the other end to
the wall. The top of the ladder is at a height
is h up the wall and the base of the ladder is
at a distance b from the wall.

Version 022 Test 4 florin (57850)

10
F

W1

d
P

W1

W2 T

W2

P ivot

Nf

b
The weight of the ladder is W1 . 
Jill, with


a weight W2 , is one-fourth the way d =


4
up the ladder. The force which the wall exerts
on the ladder is F .
What is the torque equation about P ?
b
W2 + b W1 = F h
2
b
2. (W1 + W2 ) = F h
2
h
h
3. W2 + W1 = F b
4
2
b
b
4. W2 + W1 = F h correct
4
2
h
5. W2 + h W1 = F b
2
h
6. (W1 + W2 ) = F b
2
1.

Applying rotational equilibrium,


X

P = W2 d cos +W1

cos F sin = 0 ,
2

where d is the distance of the person from the


h
bottom of the ladder, sin =
and cos =

b
.

2 F sin = 2 W2 d cos + W1 cos


db
+ W1 b
2 F h = 2 W2

b
= 2 W2 + W1 b
4
b
b
F h = W2 + W1
4
2
020 10.0 points
A sticky blob strikes and sticks to a free rod,
which is initially at rest, as shown.

Let E be the mechanical energy of the sys~ the linear momentum of the system,
tem, P
~
and L the angular momentum of the system.
What is conserved?
~ and E
1. P
~ P
~ , and E
2. L,
Explanation:

~ and E
3. L

Version 022 Test 4 florin (57850)


~ only
4. L
~ and P
~ correct
5. L
~ only
6. P
Explanation:
The mechanical energy of the system is not
conserved because this is not an elastic collision, but the linear momentum and angular
momentum are always conserved in such free
collisions.

11

Version 068 Test 4 swinney (58385)


This print-out should have 20 questions.
Multiple-choice questions may continue on
the next column or page find all choices
before answering.

and from the angular momentum principle


=

rF
.
I

From kinematics for any time t


001 10.0 points
A spool floating in space has radius r mass
m and moment of inertia about its center
I = m r 2 . The spool is unwound by a
constant force F .
F
r

vtrans =

rF
t.
I
Now equation (1) implies
rot =

Ktrans

1.
2.
3.
4.
5.
6.
7.
8.
9.
10.

Ktrans
Krot
Ktrans
Krot
Ktrans
Krot
Ktrans
Krot
Ktrans
Krot
Ktrans
Krot
Ktrans
Krot
Ktrans
Krot
Ktrans
Krot
Ktrans
Krot

1
1
2
= m vtrans
= m
2
2

(2)

F
m

2

t2 ,

(3)

while equation (2) gives


Krot

1
1
2
= I
= I trans
2
2

rF
I

2

t2 ,

(4).

Finally, dividing equation (3) by equation (4),

= (1 )2
= (1 + )

(1)

and

If initially the spool is motionless, at some


later time what is the ratio of translational
kinetic energy to rotational kinetic energy?

F
t
m

Ktrans
I/r 2
= .
=
Krot
m

= 2

002 10.0 points


A door is opened by a pulling force in the x

direction.

1
=

= correct

=1+
=

1+

In which direction is the net torque vector


~ about the hinge due to the force applied to
the door handle?

=1
=

1
2

1. y correct

Explanation:
From the momentum principle

a=

F
m

2. Insufficient information is given.


3.
x
4.
z

Version 068 Test 4 swinney (58385)


2




2
4. Nf = g
sin + 1 m cos M
5.
y
d
3




2
5. Nf = g
cos 1 m M
6.
z
d
3



1

7. x

cos + 1 m + M cor6. Nf = g
d
3
rect



Explanation:

2
7. Nf = g
sin 1 m M
d
3
~
r



~ = r F

1
8. Nf = g
sin 1 m + M
F
d
3



~r points in the z direction from the hinge to
1

~
the spot where F acts, so by the right hand
cos 1 m + M
9. Nf = g
d
3
~ points in the direction y, along the
rule, ~r F



1

hinge line.
cos + 1 m + sin M
10. Nf = g
d
3
003 10.0 points
Explanation:
A crane of mass M supports a load m. The
Since the crane and boom is in static equicranes boom is length and the angle it
librium, the net torque and net force acting
makes with the horizontal is . The distance
on it are both zero.
between the front and rear wheels is d and
The equation for net torque about the front
the center of mass of the crane plus boom is
wheel is
located a distance d/3 from the rear wheel.
The ground exerts Nr and Nf at the wheels.
2
The acceleration of gravity is g .
M g d mg cos Nr d = 0
3
2

Nr = cos mg M g
d
3
And the equation for net force is
m
M

Nf + Nr (m + M ) g = 0
Nf + Nr = (m + M ) g
Adding these two equations eliminates Nr
in favor of Nf , i.e.,

Nr

Nf

What is the magnitude of the normal force


Nf ?




1
1. Nf = g
sin + 1 m + M
d
3



2

2. Nf = g
cos + 1 m M
d
3



2

3. Nf = g
sin + 1 m M
d
3

2
Nf = (m + M ) g + cos mg M g
d
3


1
=g
cos + 1 m + M
d
3
004 10.0 points
A ball of radius r with a wire glued to one
spot on its surface can be pulled along the
floor and will slide without any tendency to
roll only if when the wire is horizontal it is
also a distance h above the floor.

Version 068 Test 4 swinney (58385)

~acm
h

~
F

In terms of F , the magnitude of the applied


force, fk , the magnitude of the force of kinetic
friction, and the radius r of the ball, what is
h?
 
F
1. h = r
f
 k

F
2. h = r 1
fk


fk
3. h = r 1
correct
F
4. h = r

005 10.0 points


The figure shows a system made of two point
masses each with mass m that rotates at an
angular speed of . The masses are connected
by massless flexible spokes of length r that can
be elongated or shortened.
y
m

r
x

m
What is the the final kinetic energy Kf in
terms of the initial kinetic energy Ki if all the
spokes lengths are decreased by a factor of 2?

5. h = r (F fk )


fk
6. h = r 1 +
F

1
Ki
4
1
Ki
2. Kf =
16
1. Kf =

7. h = 0 ; the wire must run along the floor.


8. h = r (fk F )
 
fk
9. h = r
F


F
10. h = r 1 +
fk

3. Kf = 4 Ki correct
4. None of these answers are correct.
5. Kf =

Explanation:
Taking torques about the center of mass,
for no rolling,

1
Ki
2

6. Kf = Ki
7. Kf =

1
Ki
8

8. Kf = 2 Ki

r-h

fk
0 = (r h) F r fk


fk
h=r 1
.
F

9. Kf = 8 Ki
~
F

10. Kf = 16 Ki
Explanation:
Basic Concepts:

L = I = M r2
L2 = L1

Version 068 Test 4 swinney (58385)


First the new moment of inertia is

I = 2 m r 2

= 2 m 1/2)2 r 2

= (1/4) 2 m r 2
I = (1/4) I,

Notice that rotational kinetic energy can


K
L2
I
be written K =
, hence
= since L
2I
K
I
is constant. And so for the problem at hand
K
= 4.
K
keywords:
006

10.0 points

An isolated system has 100 quanta of energy distributed between two blocks in contact. Block 1 and Block 2 have 300 and 200
quantum mechanical harmonic oscillators respectively and have q1 and q2 quanta respectively. An plot of a nonequilibrium state is
shown.
At equilibrium, what is the relationship bedS1
dS2
tween m1 =
and m2 =
?
dq1
dq2
1. m1 = m2 = 0.
2. m1 6= 0 and m2 = 0.
3. m1 = m2 . correct
4. m1 > m2 but neither are 0 or .

5. m1 = m2 = .
6. m1 = 0 and m2 6= 0.
7. m1 = and m2 is neither 0 nor .
8. m1 < m2 but neither are 0 or .
9. It is impossible to know.
10. m1 is neither 0 nor and m2 = .
Explanation:
The slope of the entropy curve gives the
temperature of each block with a given quanta
of energy in that block. When equilibrium is
reached, the temperatures, i.e., (dS/dE)1 ,
will be the same so the slopes will be the
same. Thus the correct answer is m1 = m2 .
007 10.0 points
Using the Einstein model of a solid, what
is the change in entropy when adding two
quanta of energy to a system of 5 atoms that
already has 5 quanta of energy stored in it?
(In an Einstein solid each atom corresponds
to three independent oscillators.)
 
10!
1. S = kB ln
7!


21
2. S = kB ln
75
 
17
3. S = kB ln
6
 
10
4. S = kB ln
7


20!
5. S = kB ln
7! 5!


16!
6. S = kB ln
7! 7
7. S = kB ln (10) correct


21!
8. S = kB ln
14! 7!


21!
9. S = kB ln
7! 5!

Version 068 Test 4 swinney (58385)


10. S = kB ln

20!
14! 5!

5
SB

Explanation:
Entropy is defined as S = kB ln where
is the number of possible microstates, which
is the number of ways to arrange q quanta in
n harmonic oscillators, i.e.
=

(n 1 + q)!
(n 1)! q!

(14+5)!
14!5!

008 10.0 points


A person of weight WP stands motionless
on the end of a uniform diving board of weight
WD that has two supports A and B which, if
they exert vertical forces, exert forces SA and
SB respectively. The normal force exerted on
the person is N .

3. More than one of these free body diagrams


is possible.
4.
SA

SB

WD

SB

What is the free body diagram for the diving board?

WP

5.

SA

1.

WP

14!7!

21 20
76


21 10
= kb ln
73
= kb ln (10)

SB WD

SA

S = Sf Si




(14 + 7)!
(14 + 5)!
= kb ln
kb ln
14! 7!
14! 5!
 (14+7)! 
= kb ln

WP

2.

Here, n 1 = 3 5 1 = 14 and initially


qi = 5, but in the final state qf = 7. Thus,
the change in entropy is

= kb ln 

WD

6.

WD

Version 068 Test 4 swinney (58385)


SA

SB

SB

WD

correct

WP
N

8.

SA

SB WD

SB WD

WP

WP

SA

WD

WP

could the net torque about the point B of the


diving board be zero.
009 10.0 points
The centers of mass of three trucks parked on
a hill are shown by the mark
and each of
the dotted lines are oriented vertically.
3
2
1

9.
N

Which truck(s) will tip over?


1. Trucks 1 and 2
SA

WD

WP
N

2. Trucks 2 and 3
3. Truck 2

10.
4. Truck 3
5. None of the trucks
SB WD

WP

6. All three trucks


7. Truck 1 correct

Explanation:

F = 0 and

cannot have a net transitional force of zero


since all forces act downward.
For the other three diagrams, only in
SB

7.

SA

For static equilibrium,


X
= 0.

SA

WD

Version 068 Test 4 swinney (58385)

is the product of the number of ways of arranging the energy in respective objects, i.e.
total = 1 2

8. Trucks 1 and 3
Explanation:
3
2
1

The center of gravity (CG) of Truck 1 is not


above its support base; the CGs of Trucks 2
and 3 are above their support bases. Therefore, only Truck 1 will tip.
010 10.0 points
Two objects share a total energy E = E1 +E2 .
There are 5 ways to arrange an amount of
energy E1 in the first object and 10 ways to
arrange an amount of energy E2 in the second
object. How many different ways are there to
arrange the total energy E = E1 + E2 so that
there is E1 in the first object and E2 in the
other?
1.

14!
5!9!

011 10.0 points


Two disks of identical mass but different radii
(r and 2 r) are spinning on frictionless bearings at the same angular speed 0 but in opposite directions. The two disks are brought
slowly together. The resulting frictional force
between the surfaces eventually brings them
to a common angular velocity.

0
What is the ratio of final kinetic energy Kf
to the initial kinetic energy Ki ?
1.
2.

3. 15
4. 10

3.

5. 1015

4.

14!
6.
4!10!

5.

7. 50 correct

6.

8. 5

7.

15!
5!10!

10. 105
Explanation:
The total number of microstates or number of ways of arranging energy in the system

2r

2. 225

9.

8.
9.
10.

Kf
Ki
Kf
Ki
Kf
Ki
Kf
Ki
Kf
Ki
Kf
Ki
Kf
Ki
Kf
Ki
Kf
Ki
Kf
Ki

2
5
1
=
25
4
=
25
=

=1
9
correct
25
16
=
25
1
=
5
3
=
5
=

=2
=

4
5

Version 068 Test 4 swinney (58385)

Explanation:
Note: Since the disks are spinning in opposite directions, let 1 = 0 and 2 = 0 .
The inertia of the larger disk is

1
I1 = m (2 r)2 = 2 m r 2 ,
2

2m, r

m
k

3m

and of the smaller disk

I2 =

1
m r2 .
2

Using conservation of angular momentum,

I i i = I f f
I1 0 I2 0 = (I1 + I2 ) f
I1 I2
f =
0
I1 + I2
1
2 m r2 m r2
2
0
=
1
2
2
2mr + mr
2
3
=
0 .
5

Now the initial kinetic energy is Ki =


(1/2)(I1 + I2 )02 , while the final kinetic energy is Kf = (1/2)(I1 + I2 )(3/5)202 . Hence
the ratio is just (3/5)2 = 9/25.
012 10.0 points
Two blocks are connected by a massless, inextensible string wrapped around a massive,
frictionless pulley as shown below. The block
resting on the plane which is inclined at angle from the horizontal, has mass m. The
pulley is a uniform disk and has mass 2m and
radius r. And the hanging block has mass
3m. The coefficient of kinetic friction between
block and the incline is k . The acceleration
of gravity is g .

If the system starts from rest, how long


does it take the block on the incline to slide a
distance ?
s  

5
1
1. t =
g
3 + k cos
s  

5
1
2. t =
g
3 k cos
s  

5
1
3. t =
g
3 k sin
s


10
1
4. t =
g
3 + k cos
s


10
1
5. t =
correct
g
3 k cos
s


10
1
6. t =
g
3 + k sin
s  

3
1
7. t =
g
3 + k sin
s  

3
1
8. t =
g
3 k cos
s


10
1
9. t =
g
3 k sin
s  

5
1
10. t =
g
3 + k sin
Explanation:

Version 068 Test 4 swinney (58385)


0. 5

260

8 re
v
/ mi n

Let T1 be the tension in the string attached


to the block with mass m and T2 be the tension in the string attached to the block with
mass 3m. Now the equation of motion for
block m is

T1 fk = ma
T1 k mg cos = ma.
The equation of motion for the disk with
I = mr 2 is
(T2 T1 )r = mr 2
T2 T1 = ma,
where the last equality follows from r = a.
The equation of motion for 3m is simply
3mg T2 = 3ma.
Summing the three equations of motion together eliminates the tensions and leads to

mg (3 k cos ) = 5ma
g
(3 k cos ) .
a=
5

x
0.4 m
radius

013 10.0 points


A 3 kg bicycle wheel rotating at a
2608 rev/min angular velocity has its shaft
supported on one side as shown. When viewed
from the left (from the positive x-axes), the
wheel is rotating in a clockwise manner. The
distance from the center of the wheel to the
pivot point is 0.5 m. The wheel is a hoop of
radius 0.4 m with its shaft is horizontal.

mg

Find the change in the precession angle


after a 1.2 s time interval. Assume all of the
mass of the system is located at the rim of the
bicycle wheel. The acceleration of gravity is
9.8 m/s2 .
1. 7.9628
2. 7.70981
3. 9.57486
4. 6.10127
5. 10.4337
6. 3.86619
7. 7.56292
8. 4.89373
9. 3.47894
10. 4.39085
Correct answer: 7.70981 .
Explanation:

Finally, from kinematics


1
= at2
2
s



10
1
t=
g
3 k cos

Let : m = 3 kg ,
= 2608 rev/min ,
b = 0.5 m , and
R = 0.4 m .
Viewed from Above
+x

wheel

x
L
L+ L
L

+y

The magnitude of the angular momentum


I of the wheel is L = I = m R2 and is
directed along the negative x-axis.
L
=
and
L

Version 068 Test 4 swinney (58385)

= (2608 rev/min)

2 1 min
rev 60 s

= 273.109 rad/s .
~
dL
The torque is ~ =
so L = t and
dt
the precession angle is
L
t
m g b t
=
=
=
L
L
m R2
(9.8 m/s2 ) (0.5 m) (1.2 s)
g b t
= 2 =
R
(0.4 m)2 (273.109 rad/s)
= 7.70981 .
014 10.0 points
Initially a wheel rotating about a fixed
axis with a constant angular acceleration
of 0.4 rad/s2 , has an angular velocity of
1.2 rad/s, and an angular position of 6.7 rad.
What is the angular position of the wheel
after 2.9 s?
1. 6.2
2. 8.897
3. 10.859
4. 9.15
5. 11.2
6. 11.5
7. 8.498
8. 12.4595
9. 13.572
10. 9.468
Correct answer: 8.498 rad.
Explanation:
= 0.4 rad/s2 ,
0 = 1.2 rad/s ,
0 = 6.7 rad , and
t = 2.9 s .
From kinematics,
Let :

1 2
t + 0
2
= (1.2 rad/s) (2.9 s)
1
+ (0.4 rad/s2 ) (2.9 s)2 + 6.7 rad
2
= 8.498 rad .

= 0 t +

10

015 10.0 points


A rotating uniform-density disk of radius R
and mass M is mounted in the vertical plane,
as shown in the following figure.
Clay

A
b

A lump of clay with mass m falls and sticks


to the outer edge of the wheel at the position
~ = hx0 , y0 , 0i (choice
A given by the vector A
of axes: x to the right, y up, z out of the
page; origin at the center of the axle). Just
before the impact, the clay has velocity ~v =
h0, v0 , 0i, and the disk is rotating clockwise
with angular speed 0 .
Just after the impact, what is the angular
speed f of the combined system of wheel
plus clay?


mv0 R 1/2M R20
1. f =
(1/2M + m)R2


mv0 R + 1/2M R20
2. f =
(1/2M m)R2


mv0 x0 + 1/2M R20
3. f =
(1/2M + m)R2


mv0 y0 + 1/2M R20
4. f =
(1/2M m)R2


mv0 x0 1/2M R20
5. f =
correct
(1/2M + m)R2


mv0 x0 1/2M R20
6. f =
(1/2M m)R2


mv0 R 1/2M R20
7. f =
(1/2M m)R2

Version 068 Test 4 swinney (58385)




mv0 y0 1/2M R20
8. f =
(1/2M + m)R2


mv0 y0 1/2M R20
9. f =
(1/2M m)R2


mv0 x0 + 1/2M R20
10. f =
(1/2M m)R2

Explanation:
To calculate the angular speed of the combined system, one needs to compute the moment of inertia of the system.
1
Isys = M R2 + m R2 .
2
Therefore,


~

Ltot,f
sys =
I
sys

mv0 x0 1/2M R20
=
(1/2M + m)R2
016 10.0 points
A disk of radius 9.5 cm and mass 0.52 kg is
pulled along a frictionless surface with a force
of 15 N by a string wrapped around the edge
(Fig. 11.43, displayed below). 24 cm of string
has unwound off the disk. At the instant
shown the angular velocity is 21 rad/s in the
clockwise sense.
24 cm

15 N

11

5. 0.207319
6. 0.21821
7. 0.193617
8. 0.204532
9. 0.259966
10. 0.334277
Correct answer: 0.334277 kg m2 /s.
Explanation:
~ . Therefore, |~ | = |~r ||F
~| =
Use ~ = ~r F
(0.095 m) (15 N). Thus, |~ | = 1.425 N m.
~ = I~
L
=< 0, 0, I >, where I =
2
(1/2)M R for the solid disc and
~ is directed
along the negative z-axis. Using the values
given from the beginning of the problem, the
equation becomes

~ =< 0, 0, (1/2)M R2 >


L
=< 0, 0, 0.0492765 kg m2 /s >
~ = 0.0492765 kg m2 /s
|L|
Since the torque acting on the disc is constant over time, we conclude the change in
angular momentum is given by
~ = ~ t
L
~ =< 0, 0, RF t >
L
The final angular momentum is then given by

9.5 cm

At time t = 0.2 s later, what is the magnitude of the angular momentum about the
center of the disk?
1. 0.26624
2. 0.29285
3. 0.23137
4. 0.276403

~f = L
~ + L
~
L
~ < 0, 0, RF t >
= L+

=< 0, 0, (1/2)M R2 RF t >


=< 0, 0, 0.0492765 0.285 > kg m2 /s
=< 0, 0, 0.334277 kg m2 /s >

~ f | = 0.334277 kg m2 /s
|L

017 10.0 points


A uniform beam of mass M supports two
weights that hang motionless.

Version 068 Test 4 swinney (58385)


L
4

L
4

672 N

L
2

B
W

Find W such that the supporting force at


A is zero. Assume the force acting at B has
no horizontal component.
1. 445.5
2. 351.0
3. 336.0
4. 491.0
5. 363.0
6. 342.0
7. 478.0
8. 438.5
9. 382.5
10. 378.5
Correct answer: 336 N.
Explanation:

Let :

Consider the system of the putty and the


rod, and let E be the kinetic energy of the sys~ the linear momentum of the system,
tem, P
~
and L the angular momentum of the system.
List which of these quantities are conserved,
first for a point about the axle and second for
a point about the center of the rod.
~ ; center: none
1. axle: P
2. axle: none; center: none
~ , E; center: L
~ ,E
3. axle: L
~ , E; center: L
~
4. axle: L
~ ,P
~ , E; center: L
~ ,P
~
5. axle: L
~ center: none correct
6. axle: L;
~ ,P
~ ; center: L
~ ,P
~
7. axle: L
~ ,P
~ , E; center: L
~ ,P
~ ,E
8. axle: L

W1 = 672 N and
FA = 0 .

= 0, so for a fulcrum at B,

W1

12

L
L
=W
4
2
1
1
W = W1 = (672 N) = 336 N .
2
2

018 10.0 points


A sticky ball collides with a rod exactly at the
point where the rod can rotate on a frictionless
axle as shown. The axle is permanently fixed
in place and the rod is initially at rest.

~ ,P
~ ; center: L
~
9. axle: L
~ center: L
~
10. axle: L;
Explanation:
First, notice the rod doesnt rotate or translate. The mechanical energy of the system is
not conserved because initially there was kinetic energy and finally there was not; mechanical became internal energy or was dissipated as heat. Linear momentum is not
conserved since there was motion before but
was none after; a force was exerted on the system by the axle. Finally, for the point about
the axle angular momentum is conserved since
initially it was zero and finally it was zero; the
force from the axle acted through the point of
rotation and applied no torque. And for the

Version 068 Test 4 swinney (58385)


point about the center, angular momentum is
not conserved because it was first nonzero and
then later was zero; here the force due to the
~ is conserved
axle did apply a torque. Hence L
about the axle and nothing is conserved about
the center.
019 10.0 points
There is a moon in a circular orbit around an
Earth-like planet. The mass of the moon is
8.26 1022 kg, the center-to-center separation
of the planet and the moon is 9.29 105 km,
the orbital period of the moon is 26 days, and
the radius of the moon is 1500 km.
What is the translational angular momentum of the moon about the planet?
1. 1.71642e+34
2. 5.64551e+34
3. 2.07747e+35
4. 4.57364e+33
5. 1.67004e+35
6. 2.60541e+34
7. 1.9939e+35
8. 1.26096e+35
9. 2.78853e+34
10. 3.13592e+34
Correct answer: 1.9939 1035 kg m2 /s.

Explanation:
The angular speed in radians per unit
time, for a complete circle is
=

2
.
T

Translational angular momentum Ltrans is


2 m r2
T
kg)

Ltrans = m v r = m r 2 =

= 2 (8.26 1022

2
1000
m
(9.29 105 km)
1 km



24 hr
(26 days)
1 day
35
= 1.9939 10 kg m2 /s .
020 10.0 points
A uniform rod of mass m and length is

13

pivoted about a horizontal, frictionless pin


at the end of a thin extension (of negligible
mass) a distance from the center of mass of
the rod. The rod is released from rest at an
angle of with the horizontal, as shown in the
figure.

m
O

What is the magnitude of the vertical force


Fy exerted on the rod by the pivot at the
instant the rod is in a horizontal position?
The acceleration due to gravity is g and the
moment of inertia of the rod about its center
1
m 2 .
of mass is
12
1
1. Fy = mg
5
2. Fy = mg
1
3. Fy = mg
3
1
4. Fy = mg
2
13
5. Fy = mg
12
6. Fy = 3mg
7. Fy = 2mg
1
mg correct
13
12
9. Fy = mg
13
25
10. Fy = mg
13
8. Fy =

Explanation:

Version 068 Test 4 swinney (58385)

I = ICM + m d2 =

13
1
m 2 + m 2 =
m 2 .
12
12

Fy
Fx

O
mg

90

Since the rod is uniform, its center of mass


is located at . The weight m g acts at the
center of mass, so the magnitude of the torque
is


 =rF =I
13
m 2 = m g sin 90 = m g
12
12 g
.
=
13

The vertical component of the tangential acceleration is




12 g
12
ay = r =
g
=
13
13
and the sum of the forces acting on the rod in
the vertical direction is
Fy m g = m ay ,
so the vertical reaction force Fy on the pivot
is


12
mg
Fy = m (g ay ) = m g
g =
.
13
13

14

Answer, Key { Homework 8 { Rubin H Landau


1
This print-out should have 14 questions. the center is:
Z
Check that it is complete before leaving the
Icm = r2 dr
printer. Also, multiple-choice questions may
continue on the next column or page: nd all
Z R
m
2
choices before making your selection.
=
r
r dr
2
2
(Rout , Rin )l R
We are back in order (let's hope). Some
Z 2
Z l
solutions may be found on the class home
 d dz
page.
0
0
4 , R4
R
m
out
in (2 )(l)
Rolling of a Cylinder
=
2
2
4
(Rout , Rin )l
11:01, trigonometry, numeric, > 1 min.
m
2
2
004
= 2 (Rout
+ Rin
)
An m = 3:34 kg hollow cylinder with Rin =
0:3 m and Rout = 0:5 m is pulled by a horizon- By the parallel axis theorem, the moment of
tal string with a force F = 22:8 N, as shown inertia about the ground is:
in the diagram.
2
I = Icm + mRout
2
2
+ Rin
)
= m2 (3Rout
out

in

F
Rin
Rout

What must the magnitude of the force of


friction be if the cylinder is to roll without
slipping?
Correct answer: 4:34286 N.

Explanation:

Suppose the cylinder has a length of l.


Then the density of the cylinder is  =
m
2
(Rout , Rin2 )l . The moment of inertia about

From the force equation, we have:


F + f = ma:
From the torque equation, we have:
F (2Rout) = I = I Ra :
out
Solving this pair of equations, we get:


2
2
mR
out
f =F
I , 1

2
2(3
:
34
kg)(0
:
5
m)
= 22:8 N
1:4028 kgm2 , 1
= 4:34286 N

005

In what direction is the frictional force?

1. To the left
2. To the right correct
3. Force is zero
Explanation:

The frictional force calculated has the same


sign as the applied force, so it must be in the
same direction as F , i.e. to the right.

006

Answer, Key { Homework 8 { Rubin H Landau


2
From the perspective of the ground and
What is the acceleration of the cylinder's cennoting that the acceleration is constant, the
ter of mass?
distance traveled by the disk before the pure
Correct answer: 8:1266 m=s2.
rolling occurs is,
Explanation:
From the pair of equations above, we can
solve for a:
1 a t2
2
2
FR
s
=
out
a= I
2 
2
R
!
1
2
0
2(22
:
8
N)(0
:
5
m)
= 2 g 3 g
= 1:4028 kgm2 :
R2 !02
= 8:1266 m=s2
= 18
g
Algorithm 0:1
(0:187 m)2 (19 rad=s)2
=
r=0:3 m 0:3
(1)
18 (0:0643) (9:8 m=s2)
0:4
R=0:5 m 0:6
(2)
= 1:11296 m :
2
m=3:34 kg 5
(3)
20
F =22:8 N 50
(4)
,

I =0:5 m 3:0 R2:0 +r2:0
(5) Algorithm
(1)
hcmm i = 0:01 m=cm
,

2:0
2:0
=0:5 h3:34i 3:0 h0:5i +h0:3i
2
g = 9:8 m=s 
(2)
2
=1:4028 kgm
r = 18:7 cm 10
(3)
,

2
2:0
2:0
30
hkgm i=hi hkgi hi hmi +hmi
units
10
!0 = 19 rad=s 20
(4)
2:0
0:05
2
:
0
F
R
a=
(6)
 = 0:0643 0:2
(5)
I
m
R = r hcmi
(6)
:8i h0:5i2:0
= 2:0 h22
= h18:7i h0:01i
h1:4028i
= 0:187 m
=8:1266 m=s2
hmi = hcmi hm=cmi
units
2:0
hi
h
N
i
h
m
i
2
hm=s i= hkgm2i
units
R !0
t
=
(7)


3
:
0

g
2
:
0
i h19i
f =F 2:0 mIR ,1
(7)
= 3:0hh00:187
:0643i h9:8i


2:0
2
:
0
h
3
:
34
i
h
0
:
5
i
= 1:87948 s
=h22:8i
h1:4028i ,1
=si
hsi = hhimhii hhrad
units
2i
=4:34286
N
m
=
s


i hmi2:0 ,hi
R2:0 !02:0
hNi=hNi hi hkg
units
2
s
=
(8)
hkgm i
18:0  g
h0:187i2:0 h19i2:0
=
Contacting a Surface
18:0 h0:0643i h9:8i
11:01, advanced, numeric, > 1 min.
= 1:11296 m
008
2:0
=si2:0
hmi = hmhii hihhrad
units
Determine the distance the disk travels before
2i
m
=
s
pure rolling occurs.
Correct answer: 1:11296 m.

Explanation:

009

Answer, Key { Homework 8 { Rubin H Landau


2
l2
2. 12 ms
kg  m2
3.
13
:
5
M1
s2
4.
18 Nkg m
l2
5. 9 Nkg m

l1
m
l1

M2

A rod of negligible mass is pivoted at a point


that is o -center, so that length `1 is diferent
from length `2 . the gures above show two
cases in which masses are suspended from the
ends of the rod. In each case the unknown
mass m is balanced by a known mass, M1
or M2, so that the rod remains horizontal.
What is the value of m in terms of the known
masses?

1. M1 + M2

2. M1 +2 M2
3. M1M2
p
4. M1M2 correct

5. M12M2
Explanation:

Explanation:

The angular momentum is

L = mvr
= (2 kg)(3 m=s)(4 m)
2
= 24 kg s m :

Mass on Solid Cylinder


011

11:01, calculus, numeric, > 1 min.

A 7:9 kg mass is attached to a light cord,


which is wound around a pulley. The pulley
is a uniform solid cylinder of radius 11:6 cm
and mass 1:89 kg. What is the resultant net
torque on the system about the center of the
wheel?
Correct answer: 8:98072 kg m2=s2 .

Explanation:

The balance in the rst case requires m`1 =


M1`2. And the balance in the second case
requires M2`1 = m`2. Cancel `1 and `2 from
the above
equations. So m2 = M1M2 , i.e.
p
m = M1 M2.

010

A 2 kg object moves in a circle of radius 4 m


at a constant speed of 3 m/s. A net force of
4.5 N acts on the object. What is the angular
momentum of the object with respect to an
axis perpendicular to the circle and through
its center?
2
1. 24 kg s m correct

The net torque on the system is the torque


by the external force, which is the weight of
the mass. So it is given by:

 = r F sin  = r m g sin 90 = r m g


= 0:116 m  7:9 kg  9:8 m=s2
= 8:98072 kg m2=s2:

012

When the falling mass has a speed of 5:47 m=s,


the pulley has an angular velocity of v=r.
Determine the total angular momentum of
the system about the center of the wheel.
Correct answer: 5:61233 kg m2=s.

Explanation:

The total angular momentum has two


parts, one of the pulley and one of the mass.

Answer, Key { Homework 8 { Rubin H Landau


2:0
So it is
u
I = 1:0Mr
2:0
jL~ j = j~r  m ~v + I ~!j
1:0h1:89ih0:116i2:0




=
2:0
= r m v + 12 M r2 vr = r m + M2 v
= 0:0127159 kg m2


= 11:6 cm 7:9 kg + 1:892 kg v = 1:02602 kg m v hkg m2i = hihkgihmi2:0
hi
v
= 1:02602 kg m  5:47 m=s = 5:61233 kg m2=s :
!=r
u
h5:47i
Mass on Solid Cylinder
=
11:02, calculus, multiple choice, < 1 min.
h0:116i

013

Using the fact that  = dL=dt and your result


from the previous part, calculate the acceleration of the falling mass.
Correct answer: 8:75297 m=s2.

Explanation:

Use the torque-angular momentum relation, we have


d (1:02602 kg m v) = 1:02602 kg m a ;
=
 = dL
dt dt
Solving for acceleration:

a = 1:02602
kg m
2 =s2
8
:
98072
kg
m
= 1:02602 kg m
= 8:75297 m=s2:

Algorithm
hcmm i = 0:01 m=cm

m = 7:9 kg 48

r = 11:6 cm 158
ru = rhcmm i
= h11:6ih0:01i
= 0:116 m
hmi = hcmihm=cm i
M = 1:89 kg 13
g = 9:8 m=s2
 = rumg
= h0:116ih7:9ih9:8i
= 8:98072 kg m2=s2
hkg m2=s2 i = hmihkgihm=s 2i
v = 5:47 m=s 105

(1)
(2)
(3)
(4)
units
(5)
(6)
(7)
units
(8)

4
(9)

units
(10)

= 47:1552 s,1
=si
hs,1i = hm
units
hmi
L1 = rumv
(11)
= h0:116ih7:9ih5:47i
= 5:01271 kg m2=s
hkg m2=si = hmihkgihm=si
units
L2 = I!
(12)
= h0:0127159ih47:1552i
= 0:599621 kg m2=s
hkg m2=si = hkg m2ihs,1i
units
L = L1 + L2
(13)
= h5:01271i + h0:599621i
= 5:61233 kg m2=s
hkg m2=si = hkg m2=si + hkg m2=si units
(14)
b = Lv
i
= h5h:561233
:47i
= 1:02602 kg m
2
units
hkg mi = hkghmm=s=isi
a = v
(15)
L
ih5:47i
= h8:98072
h5:61233i
= 8:75297 m=s2
2 2
hm=s2i = hkg hmkg =ms 2ih=smi =si

Child on a MerryGoRound

units

11:03, trigonometry, multiple choice, < 1 min.

Answer, Key { Homework 8 { Rubin H Landau


5
014
= h200i + h21i h2:5i2:0
A playground merry-go-round of radius 2:5 m
= 331:25 kg m2
2
has a moment of inertia 200 kg m and is
hkg m2i = hkg m2i + hkgi hmi2:0 units
rotating at 9:5 rev=min. A child with mass
21 kg jumps on the edge of the merry-go!2 = !1II1
(6)
2
round.
5i h200i
What is the new moment of inertia of the
= h9h:331
:25i
merry-go-round and child, together?
Correct answer: 331:25 kg m2.
= 5:73585 rev=min
Explanation:
i hkg m2i units
hrev=mini = hrev=min
The moment of inertia will be the combihkg m2i
nation of the individual moments of inertia of
each component.

018

Imerry,go,round + Ichild = Itotal

Child on a MerryGoRound
11:03, calculus, numeric, > 1 min.
015

Assuming that the boy's initial speed is negligible, what is the new angular speed of the
merry-go-round?
Correct answer: 5:73585 rev=min.

A bicycle wheel of mass m rotating at an


angular velocity ! has its shaft supported
on one side, as shown in the gure. When
viewing from the left, one sees that the wheel
is rotating in a counterclockwise manner. The
distance from the center of the wheel to the
pivot point is b. We assume the wheel is a
hoop of radius R, and the shaft is horizontal.
b

Explanation:
Basic Concepts:
X

L~ = const

The net angular momentum of the system remains constant, therefore, from conservation
of the angular momentum we have:
I1 !1 = (I1 + m R2) !2
And
1
!2 = !1 I +Im
R2
1
2)
(9
:
5
rev
=
min)

(200
kg
m
= (200 kg m2) + (21 kg) (2:5 m)2
= 5:73585 rev=min

Algorithm

R = 2:5 m 12::55

I1 = 200 kg m2 100
300

!1 = 9:5 rev=min 155

m = 21 kg 20
35
I2 = I1 + m R2:0

(1)
(2)
(3)
(4)
(5)

W=mg

The magnitude of the angular momentum


of the wheel is given by
1. 14 m R2 !

2. m R2 !2
3. 12 m R2 !2
4. 14 m R2 !2

Answer, Key { Homework 8 { Rubin H Landau

5. 12 m R2 !

Precession
020

11:05, calculus, numeric, > 1 min.


The direction of precession as viewed from the
top is:

6. m R2 ! correct
Explanation:
Solution: Basic Concepts:
~
~ = dL
Top view

1. along the direction of rotation of the


wheel

dt

L
L

L+ L

The magnitude of the angular momentum of


the wheel, L, is
L = I ! = m R2 !;
since the moment of inertia of the wheel, I , is
m R2 .

019

Given: the mass 3 kg, the angular velocity


15 rad=s, the axil length b = 0:5 m, and
the radius of the wheel R = 0:49 m. Find the
precession angle in the time interval t = 1:1 s.
Correct answer: 85:7488  .

Explanation:

From the gure below, we get  = LL .


Using the relation, L =  t, where  is the
magnitude of the torque, mg  b, we get
 = LL
=  Lt
= mmgRb2!t
= gRb2!t
2
:5 m)(1:1 s)
= (9:(08 :m49=sm))(0
2 (15 rad=s)
= 1:4966 rad
= 85:7488:

2. counterclockwise correct
3. clockwise
4. opposite to the direction of rotation of the
wheel
Explanation:

From the gure, we can see the direction of


precession is counterclockwise.

Algorithm
hdeg
radi = 57:2958 deg=rad

g = 9:8 m=s
m = 3:0 kg 
! = 15 rad=s 10
0:4 15
b = 0:5 m 0:6

R = 0:49 m 00::46

t = 1:1 s 12
 = Rgbt
2:0 !
ih0:5ih1:1i
= h9h0:8:49
i2:0 h15i
= 1:4966 rad
m=s2ihmihsi
hradi = hhm
i2:0hrad=si
deg = hdeg
radi
= h1:4966ih57:2958i
= 85:7488
hi = hradihdeg=radi
2

(1)
(2)
(3)
(4)
(5)
(6)
(7)
(8)

units
(9)
units

Version 001 circular and gravitation holland (2383)


This print-out should have 19 questions.
Multiple-choice questions may continue on
the next column or page find all choices
before answering.
AP B 1993 MC 57
001 10.0 points
Two objects of masses 14 kg and 35 kg are
hung from the ends of a stick that is 70 cm
long and has marks every 10 cm, as shown.
ABCDE F G

10 20 30 40 50 60
14 kg

35 kg

If the mass of the stick is negligible, at


which of the points indicated should a cord be
attached if the stick is to remain horizontal
when suspended from the cord?

(m1 + m2 ) x = m2
(35 kg) (70 cm)
m2
=
x=
m1 + m2
14 kg + 35 kg
= 50 cm .

Therefore the point should be point G .


AP B 1998 MC 7
002 10.0 points
Three forces act on an object.
If the object is in translational equilibrium,
which of the following must be true?
I. The vector sum of the three forces must
equal zero;
II. The magnitude of the three forces must
be equal;
III. The three forces must be parallel.
1. I only correct
2. I, II and III

1. F

3. II only

2. A

4. II and III only

3. B

5. I and III only

4. C
5. E
6. G correct
7. D
Explanation:
Let : = 70 cm ,
m1 = 14 kg , and
m2 = 35 kg .
For static equilibrium, net = 0.
Let x be the distance from the left end of
the stick to the point of attachment of the
cord:

Explanation:
If an object is in translational equilibrium,
the vector sum of all forces acting on it must
equal zero.
AP M 1993 MC 35 A
003 10.0 points
A rod of negligible mass is pivoted at a point
that is off-center, so that length 1 is different
from length 2 . The figures show two cases in
which masses are suspended from the ends of
the rod. In each case the unknown mass m is
balanced by a known mass M1 or M2 so that
the rod remains horizontal.
1
2
m

T = m1 g x m2 g ( x) = 0

M1

Version 001 circular and gravitation holland (2383)


1

Which of the following expresses the condition required for the system to be in static
equilibrium?

2
m

M2

What is the value of m in terms of the


known masses?
1. m =

1. b2 m1 = a2 m2
2. a m2 = b m1
3. a2 m1 = b2 m2

M1 + M2
2

4. a m1 = b m2 correct

2. m = M1 M2
p
3. m = M1 M2 correct

5. m1 = m2

4. m = M1 + M2
M1 M2
2
Explanation:
X
Applying
= 0 to balance the masses
in both cases,

Explanation:
In equilibrium, the total torque is zero,
which gives

5. m =

m 1 = M1 2
M2 1 = m 2 .

and

a m1 = b m2 .
AP B 1993 MC 8
005 10.0 points
Two spheres have equal densities and are subject only to their mutual gravitational attraction.

Dividing,
m
M1
=
M2
m
2
m =p
M1 M2
m = M1 M2 .

Which quantity must have the same magnitude for both spheres?
1. displacement from the center of mass

AP M 1998 MC 30
004 10.0 points
Consider the wheel-and-axle system shown
below.

2. gravitational force correct


3. acceleration
4. velocity
5. kinetic energy

b
a

m1

m2

Explanation:
Two spheres with the same density have
different masses due to their relative sizes.
Using Newtons third law, F~1 = F~2 .
All of the other quantities (acceleration, velocity, kinetic energy, and displacement from
the center of mass) have different magnitudes
because the two spheres have different masses.

Version 001 circular and gravitation holland (2383)


AP M 1998 MC 7 8
006 (part 1 of 2) 10.0 points
A ball is tossed straight up from the surface
of a small, spherical asteroid with no atmosphere. The ball rises to a height equal to the
asteroids radius and then falls straight down
toward the surface of the asteroid.
What forces, if any, act on the ball while it
is on the way up?
1. Only a decreasing gravitational force that
acts downward correct
2. Only a constant gravitational force that
acts downward
3. Both a constant gravitational force that
acts downward and a decreasing force that
acts upward
4. Only an increasing gravitational force
that acts downward
5. No forces act on the ball.
Explanation:
There is no friction in the system, and the
ball doesnt have any contact with other objects, so the only force acting on the ball is
the attractive gravitational force, which acts
downward.
~ = G M m r , the force will deFrom F
r2
crease as the ball rises.
007 (part 2 of 2) 10.0 points
The acceleration of the ball at the top of its
path is
1. zero.
2. equal to one-fourth the acceleration at the
surface of the asteroid. correct
3. equal to one-half the acceleration at the
surface of the asteroid.
4. equal to the acceleration at the surface of
the asteroid.

5. at its maximum value for the balls


flight.
Explanation:
1
1
F = m a 2 , so a 2 and
r
r
1
1 1
1
a
=
a.
2
2
(2 r)
4r
4
Weight of Spacecraft in Space
008 (part 1 of 2) 10.0 points
The radius of Earth is about 6440 km. A
7770 N spacecraft travels away from Earth.
What is the weight of the spacecraft at a
height 6440 km above Earths surface?
Correct answer: 1942.5 N.
Explanation:
Let :

rE = 6440 km ,
W = 7770 N , and
h = 6440 km .

By Newtons Universal Law of Gravitation


W

1
,
r2

so

1
rh2
r2
Wh
=
= 2
1
W
rh
r2
r2
Wh = W 2
rh
2
rE
=W
(rE + h)2

= (7770 N)

(6440 km)2
(6440 km + 6440 km)2

= 1942.5 N .
009 (part 2 of 2) 10.0 points
What is the weight 53400 km above Earths
surface?
Correct answer: 89.9932 N.

Version 001 circular and gravitation holland (2383)


Explanation:
Let :
W2 = W
=W

C) The projectile undergoes gratitational


acceleration.
h = 53400 km .

r2
rh2
2
rE
(rE + h)2

(6440 km)2
= (7770 N)
(6440 km + 53400 km)2
= 89.9932 N .
AP B 1993 MC 1
010 10.0 points
Consider the following situations.
A) An object moves in a straight line at constant speed.
B) An object moves with uniform circular
motion.
C) An object travels as a projectile in a
gravitational field with negligible air resistance.
In which of the situations would the object
be accelerated?
1. C only
2. A only
3. B and C only correct

AP B 1993 MC 6
011 10.0 points
If Spacecraft X has twice the mass of Spacecraft Y , then what is true about X and Y ?
I) On Earth, X experiences twice the gravitational force that Y experiences;
II) On the Moon, X has twice the weight of
Y;
III) When both are in the same circular orbit,
X has twice the centripetal acceleration
of Y .
1. III only
2. I only
3. I and II only correct
4. II and III only
5. I, II, and III
Explanation:
I) gravitational force mass.
II) weight mass.
III) The centripetal acceleration is determined by
ac =

v2
,
r

4. B only
5. A and C only
6. None exhibits acceleration.
7. A and B only
8. All exhibit acceleration.
Explanation:
A) The velocity of the object (its direction
and magnitude) is unchanged, so it is not
accelerated.
B) The direction of the velocity constantly
changes; the centripetal acceleration is directed toward the center of the motion.

so X and Y should have the same centripetal


acceleration when they are in the same circular orbit.
AP M 1998 MC 14 15
012 (part 1 of 2) 10.0 points
A spring has a force constant of 593 N/m and
an unstretched length of 6 cm. One end is
attached to a post that is free to rotate in the
center of a smooth table, as shown in the top
view below. The other end is attached to a
4 kg disk moving in uniform circular motion
on the table, which stretches the spring by
4 cm.
Note: Friction is negligible.

Version 001 circular and gravitation holland (2383)

593 N/m
4 kg
10 cm

What is the centripetal force Fc on the disk?

AP B 1993 MC 48
014 10.0 points
The planet Krypton has a mass of
8.5 1023 kg and radius of 4 106 m.
What is the acceleration of an object in free
fall near the surface of Krypton? The gravitational constant is 6.6726 1011 N m2 /kg2 .
Correct answer: 3.54482 m/s2 .
Explanation:

Correct answer: 23.72 N.


Explanation:
Let : r = 6 cm = 0.06 m ,
r = 4 cm = 0.04 m ,
m = 4 kg , and
k = 593 N/m .
The centripetal force is supplied only by
the spring. Given the force constant and the
extension of the spring, we can calculate the
force as
Fc = k r
= (593 N/m) (0.04 m)
= 23.72 N .
013 (part 2 of 2) 10.0 points
What is the work done on the disk by the
spring during one full circle?
1. W = 0 J correct
2. W = 0.00512352 J
3. W = 4.47111 J
4. W = 1.4232 J
5. W = 8.94223 J
Explanation:
Since the force is always perpendicular to
the movement of the disk, the work done by
the spring is zero .

M = 8.5 1023 kg ,
R = 4 106 m , and
G = 6.6726 1011 N m2 /kg2 .

Let :

Near the surface of Krypton, the gravitation force on an object of mass m is


F =G

Mm
,
R2

so the acceleration a of a free-fall object is


a = gKrypton =

F
m

M
R2
= (6.6726 1011 N m2 /kg2 )
8.5 1023 kg

(4 106 m)2

=G

= 3.54482 m/s2 .

AP B 1998 MC 39
015 10.0 points
An object has a weight W when it is on the
surface of a planet of radius R.
What will be the gravitational force on the
object after it has been moved to a distance
of 4 R from the center of the planet?
1. F = 4 W
2. F =

1
W
4

3. F = W

Version 001 circular and gravitation holland (2383)


1
4. F =
W correct
16
5. F = 16 W

Explanation:
On the surface of the planet,
GM m
W=
.
R2
When the object is moved to a distance 4 R
from the center of the planet, the gravitational
force on it will be
GM m
F =
(4 R)2
GM m
=
16 R2
1 GM m
=
16 R2
1
=
W .
16
AP M 1993 MC 22
016 10.0 points
A newly discovered planet has twice the mass
of the Earth, but the acceleration due to gravity on the new planets surface is exactly the
same as the acceleration due to gravity on the
Earths surface.
What is the radius Rp of the new planet in
terms of the radius R of Earth?
1
1. Rp = R
2
2. Rp = 4 R
3. Rp = 2 R

2
4. Rp =
R
2

5. Rp = 2 R correct
Explanation:
From Newtons second law and the law of
universal gravitation, the gravitational force
near the surface is
Mm
Fg = m g = G 2
r
GM
g= 2 .
r

Mp = 2 Me and gp = ge , so
G Mp
G Me
2 G Me
=
=
2
2
R
Rp
Rp2
1
2
= 2
2
R
Rp

Rp = 2 R .

Gravity on Planet X short


017 10.0 points
Planet X has a mass 4.6 times that of the
Earth and a radius 2.51 times the radius of
the Earth.
What is the ratio of the acceleration due
to gravity on the surface of Planet X to the
acceleration due to gravity on the surface of
the Earth?
Correct answer: 0.730147.
Explanation:

Let :

MX = 4.6 ME and
RX = 2.51 RE .

The acceleration due to gravity is


a=

M
GM
2,
2
R
R

so

R 2 MX
gX
= E2
gE
RX ME

2 

gX
RE
4.6 ME
=
gE
2.51 RE
ME
= 0.730147 .

Gravity on Ceres
018 (part 1 of 2) 10.0 points
The asteroid Ceres has a mass 6.601 1020 kg
and a radius of 476.9 km.
What is g on the surface? The value
of the universal gravitational constant is
6.67259 1011 N m2 /kg2 .

Version 001 circular and gravitation holland (2383)


Correct answer: 0.193664 m/s2 .
Explanation:
Let :

M = 6.601 1020 kg ,
r = 476.9 km , and
G = 6.67259 1011 N m2 /kg2 .

The weight on the surface for an object of


mass M is
Mm
r2
M
g=G 2
r
= (6.67259 1011 N m2 /kg2 )

2
6.601 1020 kg 1 km

(476.9 km)2
1000 m

W = mg = G

= 0.193664 m/s2 .
019 (part 2 of 2) 10.0 points
How much would an 81.9 kg astronaut weigh
on this asteroid?
Correct answer: 15.8611 N.
Explanation:
The weight of the astronaut will be
W = mg
= (81.9 kg) (0.193664 m/s2 )
= 15.8611 N .

=
14 + 9 !
14 + 6 !
= ln
ln
14! 9!
14! 6!
14 + 9 !
= ln 14! 9!
14 + 6 !
14! 6!
23!
14! 6!
= ln

14! 9!
20!
23 22 21 20 19 18 17 26 15 14 13
= ln
(14 13 ) 9!
14 13 6!

20 19 18 17 16 15 14 13
23 22 21 20 19 18 17 16 15 (6!)
= ln
20 19 18 17 16 15 (9 8 7 6)
23 22 21
= ln
987
23 11 3
= ln
941
23 11 1
= ln
143
23 11
= ln
43
23 11
= ln
12

l
0

= 1 2 + 3

= 0

0
1 2 3
= 3 + 1
= (3 + 1 )

=
=

3
=
0 5
2
=
02

3
=

9
=
25

002 10.0 points


A ball of putty with mass m falls vertically
onto the outer rim of a hori zontal turntable
of radius R and moment of inertia Io that is
rotating freely with angular sp eed w i about
its vertical axis.
What is the post -collision angular speed of
the turntable plus putty?
1. Wf

Wi
=

l +m R
Io
E xplanatio n:
The final rotational inertia of t he turntableplus-putty is

004 10.0 points


A spool floating in sp ace has radius r m ass
m and m om ent of inertia ab out its center
I = (3m r 2
The sp ool is unwound by a
constant force F .
F

0-- -~

If initially the spool is motionless, at some


later time what is the r atio of translational
kinetic en ergy t o r otational kinetic energy?

1.

K trans = (3
K rot

ince tlwr<' is no <'xt<'rna l torque on the sy5t<'m


of the putty plus the turntable, we know L 1 =
L;

Expla nation:
From the moment um principle

a=-

m
and from the angular momentum principle

l ow,

WJ= - fJ

rF

lowi

a: = [

-=--- -=..,
- l o + m R2

wI -

From kinematics for any timet

w,

m R2

1+ - -

(1)

=It.

(2)

Vtrans = -

Io

and

rF

Wrot

003
10.0 points
A uniform bar of m ass M and length f is
propped against a very slick vertical wall as
shown. The angle b etween the wall and the
upper end of the bar is B. The force of static
friction between the upper end of the bar and
the wall is negligible, but the bar remains at
rest (in equilibrium).

Now equation (l ) implies

2 - 1 (p)2
2
m

- l

2 mvtrans -

trans -

t ,

2m

(3)

while equation (2) gives


_ 1

K rot -

2
_
Wtrans -

(r F)

21 I I

2
2

t ,

(4).

Finally, dividing equation (3) by equation ( 4),


2

I<t rans = I /r = {3
K rot

If we take the pivot point to be at the


middle of the rod, which expression below is
z-component of the net torque, wh ere p ositive
x is along the floor to the right and p ositive y
is upward along the wall?
R
5. 2 (Fw cos B + f s cos B - n sinB)

E x pla nat ion :


Taking the piYot at the center of rhe hnr.
the torq ue d ue to
Frr is ( i/ 2) F rr. cos fJ. t he
torque d ne to f., is ( I / 2) f, co,; fJ . and the
torque d ue to 11 is (l/ 2)n sin 0.

Con sider the system of the putty and the


rod, ~d let E b e the kinetic energy of the system , ..f the linear momentum of the system ,
and L the angular momentum of the syst em .
List which of these quantiti es are conserved ,
first for a point ab out the axle and second for
a point about the center of the rod.
6. axle:

= Iow;.

f JWJ = lowo

005 10.0 points


A sticky ball collides with a rod exactly at the
point where the r od can rot at e on a fricti onless
axle as shown. The axle is permanently fixed
in place and the rod is initially at rest.

L;

center: n one

Explanation:
First , n otice the rod doesn 't r otate or t ranslate. The m echanical en ergy of the system is
not conser ved because initially there was kinetic energy and finally t here was not; mechanical b ecame internal energy or was dissipated as heat. Linear momentum is not
con ser ved since there was m otion before but
was n one aft er ; a force was exerted on the syst em by the axle. Finally, for the point ab out
the axle angular mom entum is conser ved since
initially it was zero and finally it was zero; the
force from the axle acted through the p oint of
rotation and applied n o torque. And for the
point about the center, angular momentum is
n ot con ser ved because it was first nonzero and
then lat er was zero; h er e the force due t o the
axle did apply a t orque. Hence Lis conserved
about the axle and nothing is conser ved about
the center .
006
10.0 points
A small di sk with radius a, is coaxial with
a large disk of ra dius of b. Three for ces of
magnitudes H, F2, and Fs act tangentially on
the small and large di sks as sh own. The force
of m agnitude F2 a cts B b elow the h orizontal.

Fl

Find the m a gnitude of the n et t orque on


the syst em with r espect t o t he center of the
axle.

10.

= laF2 - bH- bFsl

Expla n ation:
T he angle doesn't matter because t he radial dj p lacement hom t he center of the circle
is perpendicular to any tangent force. And
the torque hom F2 acting at a distan ce a is
opposite the direction of the torques hom F1
and F3 acting at a distance b.

The m agnitude of the total t orque is


T

= laF2 - bFi

-b F3I

E=q

Vous aimerez peut-être aussi